Đến nội dung

Gachdptrai12 nội dung

Có 274 mục bởi Gachdptrai12 (Tìm giới hạn từ 06-05-2020)



Sắp theo                Sắp xếp  

#632064 ĐỀ THI OLYMPIC CHUYÊN KHOA HỌC TỰ NHIÊN 2016

Đã gửi bởi Gachdptrai12 on 09-05-2016 - 12:34 trong Thi HSG cấp Tỉnh, Thành phố. Olympic 30-4. Đề thi và kiểm tra đội tuyển các cấp.

Tiếp cận bằng dồn biến
Đặt $f(a,b,c)=\dfrac{a(a+1)}{(2a+1)^2}+\dfrac{b(b+1)}{(2b+1)^2}+\dfrac{c(c+1)}{(2c+1)^2}$
Ta sẽ chứng minh $f(a,t,t) \leq \dfrac{9}{16}$ với $t$ thỏa mãn $2at^2+2at+t^2=1$
Tức là chứng minh $\dfrac{a(a+1)}{(2a+1)^2}+2\dfrac{t(t+1)}{(2t+1)^2} \leq \dfrac{9}{16}$
$$<=> f(t)=\dfrac{1-t^2}{4}+2\dfrac{t(t+1)}{(2t+1)^2} \leq \dfrac{9}{16}$$
Xét $f'(t)=\dfrac{-t}{2}+\dfrac{2}{(2t+1)^3}=0<=>(2t-1)(4t^3+8t^2+7t+4) =0<=>t=0,5$
Dùng bảng biến thiên dễ suy ra $f(t) \leq f(0,5)=\dfrac{9}{16}$
Tiếp theo ta sẽ chứng minh $f(a,b,c) \leq f(a,t,t)$ với $t =\sqrt{bc}(1)$
Khi đó nếu $(1)$ đúng thì ta có thể đưa về trường hợp trên và ta có đpcm
$$f(a,b,c) \leq f(a,t,t)$$
$$<=>\dfrac{b(b+1)}{(2b+1)^2}+\dfrac{c(c+1)}{(2c+1)^2} \leq \dfrac{2\sqrt{bc}(\sqrt{bc}+1)}{(2\sqrt{bc}+1)^2}$$
$$<=>(y-z)^2.\dfrac{-y^3z^3+(y^2+yz+z^2)-4y^2z^2(y^2+yz+z^2)-4yz(y^2+yz+z^2)-3yz+1}{(2y^2+1)^2(2z+1)^2(2yz+1)^2} \leq 0(b=y^2,c=z^2)(2)$$
Đặt $m=y^2+yz+z^2$,$n=yz$,$m \geq 3n$
$$(2)<=>m(1-4n^2-4n)-4n^3-2n+1 \leq0$$
Do $2abc+ab+ac+bc=1$ nên không mất tổng quát ta có thể giả sử $bc \geq \dfrac{1}{4}=>n \geq \dfrac{1}{2}=>1-4n^2-4n<0$
Suy ra $m(1-4n^2-4n)-4n^3-2n+1 \leq -16n^3-12n^2+n+1=g(n) \leq 0$ với $1 >n \geq \dfrac{1}{2}$. Điều này đúng do $g(n)$ nghịch biến trên tập xác định

anh em thấy cách dồn này sao sao ấy vì anh đặt t=√bc thì cái đk đầu bài đâu có bằng đk dồn biến và ta thấy rõ là 2at khác với ab+ac



#631524 Chứng minh Các BĐT đa thức bậc 4 ba biến thực trên máy tính

Đã gửi bởi Gachdptrai12 on 05-05-2016 - 23:19 trong Tài liệu, chuyên đề, phương pháp về Bất đẳng thức

Cái này là đẳng thức em khai triển ra là thấy ngay điều phải chứng minh. :)

vậy em cũng cảm ơn hôm bữa tải 1 cái file anh Cẩn zô coi thì thấy là có chungứ minh cho bổ dè này :))



#631122 $\sum \sqrt{\frac{a}{a+b}}...

Đã gửi bởi Gachdptrai12 on 03-05-2016 - 23:33 trong Bất đẳng thức - Cực trị

4) một bđt mạnh hơn bđt 1) cho a,b,c thỏa ab+bc+ca>0 chứng minh

$\frac{\sqrt{a}+\sqrt{b}}{\sqrt{a+b}}+\frac{\sqrt{b}+\sqrt{c}}{\sqrt{b+c}}+\frac{\sqrt{c}+\sqrt{a}}{\sqrt{c+a}} \ge 3+\left(12-6\sqrt{2}\right)\sqrt{\frac{abc}{(a+b)(b+c)(c+a)}}$



#631120 $\sum \sqrt{\frac{a}{a+b}}...

Đã gửi bởi Gachdptrai12 on 03-05-2016 - 23:30 trong Bất đẳng thức - Cực trị

ta có 1 bài toán khác 

3) cho a,b,c không âm thỏa $ab+bc+ca> 0$ chứng minh

$\sqrt{\frac{a}{b+c}}+\sqrt{\frac{b}{c+a}}+\sqrt{\frac{c}{a+b}}\geq 2+(6-4\sqrt{2})\sqrt{\frac{abc}{(a+b)(b+c)(c+a)}}$




#631117 $\sum \sqrt{\frac{a}{a+b}}...

Đã gửi bởi Gachdptrai12 on 03-05-2016 - 23:25 trong Bất đẳng thức - Cực trị

 

cho a,b,c là các số thực không âm thỏa ab+bc+ca>0 chứng minh

$\sqrt{\frac{a}{a+b}}+\sqrt{\frac{b}{b+c}}+\sqrt{\frac{c}{c+a}}\geq 1+(6-2\sqrt{2})\sqrt{\frac{abc}{(a+b)(b+c)(c+a)}}$

2)

$2+(6-4\sqrt{2})\sqrt{\frac{abc}{(a+b)(b+c)(c+a)}}\geq \sqrt{\frac{a}{a+b}}+\sqrt{\frac{b}{b+c}}+\sqrt{\frac{c}{c+a}}$

 

bài 2 là thế này

áp dụng bđt C-S ta có 

${\left(\sum \sqrt{\frac{a}{a+b}}\right)^2 \le \left[ \sum (a+c)\right] \left[\sum \frac{a}{(a+b)(a+c)}\right] =4\left[1+\frac{abc}{(a+b)(b+c)(c+a)}\right].}$
từ đây đặt 
$x=\sqrt{\frac{2abc}{(a+b)(b+c)(c+a)}},\, 0 <x \le \frac{1}{2}.$ ta cần chứng minh 
$4+2x^2\le \left[2+\left(3\sqrt{2}-4\right) x\right]^2$ 
bđt này tương đương với 
$2+x^2 \le \left[1+\left(3-2\sqrt{2}\right)x\right]^2=1+2\left(3-2\sqrt{2}\right)x+\left(17-12\sqrt{2}\right)x^2,$
tương đương 
$2\left(3-2\sqrt{2}\right)x(1-2x) \ge 0$ hiển nhiên đúng 
 
 
 
 



#631087 Chứng minh Các BĐT đa thức bậc 4 ba biến thực trên máy tính

Đã gửi bởi Gachdptrai12 on 03-05-2016 - 22:12 trong Tài liệu, chuyên đề, phương pháp về Bất đẳng thức

Thật ra ta có kết quả sau: Với $a,b,c$ là ba số thực bất kỳ và $A,B,C$ là các số thực cho trước. Xét đa thức

\[P(a,b,c) = \sum a^4 + A\sum b^2c^2 + B \sum a^3b + C\sum ab^3 -(1+A+B+C)abc(a+b+c).\]

Khi đó nếu $3(1+A) = B^2 + BC + C^2,$ thì \[P(a,b,c) = \frac{1}{18} \sum \left [ 3a^2-3b^2+(B-C)ab-(2B+C)bc+(B+2C)ca \right ]^2 \geqslant 0.\]

Với $A=0,B=1,C=-2$ ta được bài toán 1.

Với $A=-\frac{9}{25},B=-\frac{4}{5},C=-\frac{4}{5}$ ta được bài toán 3.

Với $A=2,B=-3,C=0$ ta được ví dụ 1.

anh Huyện gửi em cái chứng minh zô thi hay ktra gì đó bí quá lôi cái này ra luôn :v :v 




#630751 $\frac{2x-x^{2}}{2x^{2}-2x+1...

Đã gửi bởi Gachdptrai12 on 02-05-2016 - 13:13 trong Bất đẳng thức - Cực trị

Ta có $2x^2-2x+1=(9x^2+1)-7x^2-2x\ge 6x-7x^2-2x=4x-7x^2$

Khi đó $\sum\frac{2x-x^2}{2x^2-2x+1}\le \sum\frac{2x-x^2}{4x-7x^2}=\sum\frac{2-x}{4-7x}$

Ta đi chứng minh:

$\sum\frac{2-x}{4-7x}\le 3\iff \sum\frac{x-2}{7x-4}\le 3\iff \sum\frac{6x-2}{7x-4}\ge 0$

Xét $f(t)=\frac{6t-2}{7t-4}$

Ta có: $f'(t)=\frac{-10}{(7t-4)^2}\le 0$

Từ đây suy ra f nghịch biến trên (0;1) (do x,y,z thuộc (0;1))

Không mất tính tổng quát giả sử: $x\le y\le z=> x\le \frac{1}{3}$

Ta cần CM: $ \sum\frac{6x-2}{7x-4}\ge 0$ hay ta đi chứng minh: $P=f(x)+f(y)+f(z)\ge 0$

Thật vậy:

Do f nghịch biến trên R và $x\le \frac{1}{3}$ nên $P\ge 3f(x)\ge 0$

Từ đây ta có điều phải chứng minh. Dấu = xảy ra khi $x=y=z=\frac{1}{3}$

ta có phân tích
${\sum \frac{2x-x^2}{2x^2-2x+1}-3 =\frac{4(x-y)^2(y-z)^2(z-x)^2}{\prod (2x^2-2x+1)} -\sum \frac{(y-z)^2(y+z-x)^2}{(2y^2-2y+1)(2z^2-2z+1)},}$ 
từ đây ta chứng minh
 
$\sum \frac{(y-z)^2(y+z-x)^2}{(2y^2-2y+1)(2z^2-2z+1)} \ge \frac{4(x-y)^2(y-z)^2(z-x)^2}{\prod (2x^2-2x+1)}.$
giả sử $x\geq y\geq z$
ta chứng minh bđt mạnh hơn
$\frac{(x-y)^2(x+y-z)^2}{(2x^2-2x+1)(2y^2-2y+1)} \ge \frac{4(x-y)^2(y-z)^2(z-x)^2}{\prod (2x^2-2x+1)}$
bđt tương đương
$(2z^2-2z+1)(x+y-z)^2 \ge 4(x-z)^2(y-z)^2.$
ta có 
$(x+y-z)^2 \ge (x+y-2z)^2 \ge 4(x-z)(y-z) \ge 0$
và 
$2z^2-2z+1=z^2+(x+y)^2 \ge (x+y)^2 \ge (x+y-2z)^2 \ge 4(x-z)(y-z) \ge 0,$
từ đây ta có đpcm
 
 
 
 



#630732 $\frac{2x-x^{2}}{2x^{2}-2x+1...

Đã gửi bởi Gachdptrai12 on 02-05-2016 - 10:38 trong Bất đẳng thức - Cực trị

Bạn nào có tài liệu về BDT hay để thi vào trường chuyên khôn

có quyển sách PP giải toán BĐT và cực trị cho HS8,9 của Vqbc ĐÓ BẠN




#630680 $\frac{2x-x^{2}}{2x^{2}-2x+1...

Đã gửi bởi Gachdptrai12 on 02-05-2016 - 00:03 trong Bất đẳng thức - Cực trị

x,y,z dương thỏa x+y+z=1

$\frac{2x-x^{2}}{2x^{2}-2x+1}+\frac{2y-y^{2}}{2y^{2}-2y+1}+\frac{2z-z^{2}}{2z^{2}-2z+1}\leq 3$

 



#630679 $\sum \sqrt{\frac{a}{a+b}}...

Đã gửi bởi Gachdptrai12 on 01-05-2016 - 23:46 trong Bất đẳng thức - Cực trị

cho a,b,c là các số thực không âm thỏa ab+bc+ca>0 chứng minh

$\sqrt{\frac{a}{a+b}}+\sqrt{\frac{b}{b+c}}+\sqrt{\frac{c}{c+a}}\geq 1+(6-2\sqrt{2})\sqrt{\frac{abc}{(a+b)(b+c)(c+a)}}$

2)

$2+(6-4\sqrt{2})\sqrt{\frac{abc}{(a+b)(b+c)(c+a)}}\geq \sqrt{\frac{a}{a+b}}+\sqrt{\frac{b}{b+c}}+\sqrt{\frac{c}{c+a}}$



#630219 $\sum \frac{1}{4a^{2}-ab+4b^{2...

Đã gửi bởi Gachdptrai12 on 29-04-2016 - 20:17 trong Bất đẳng thức - Cực trị

Cho $a,b,c$ không âm. Chứng minh rằng: 

$\sum \frac{1}{4a^{2}-ab+4b^{2}}\geq \frac{9}{7(a^{2}+b^{2}+c^{2})}$




#629476 Đề thi học sinh giỏi môn toán khối 10 khu vực DUYÊN HẢI VÀ ĐỒNG BẰNG BẮC BỘ n...

Đã gửi bởi Gachdptrai12 on 25-04-2016 - 09:59 trong Thi HSG cấp Tỉnh, Thành phố. Olympic 30-4. Đề thi và kiểm tra đội tuyển các cấp.

Bạn chứng minh đi

cái này có trong bđt hiện đại dựa vào 1 bđt chặt là $\left | (a-b)^{2}(b-c)^{2} (c-a)^{2}\right |$
P.s:đây là một bài thảo luận nên mình ko muốn đôi co  :icon6:  :icon6:  nhìu




#629471 Đề thi học sinh giỏi môn toán khối 10 khu vực DUYÊN HẢI VÀ ĐỒNG BẰNG BẮC BỘ n...

Đã gửi bởi Gachdptrai12 on 25-04-2016 - 09:06 trong Thi HSG cấp Tỉnh, Thành phố. Olympic 30-4. Đề thi và kiểm tra đội tuyển các cấp.

Dễ thấy của bạn à ._.

Mình nghĩ sao bạn biết được là hàm đơn điệu theo $r$ khi trên tử vẫn là $x^3y+y^3z+z^3x $

Cái này vẫn viết được theo dạng pqr nếu giả sử $a \geq b \geq c $ hoặc $a \geq c \geq b $

Nên bài giải sai

Vả lại

Cái kiến thức đó thi học sinh giỏi không được phép sử dụng mà phải chứng minh lại 

$x^3y+y^3z+z^3x $ nó vẫn viết được duois dạng pqr nhưng vẫn là biểu thức bậc 1 theo bậc 2 theo r :v :v   còn cách chứng minh của cái này khá ngắn  :closedeyes:




#629427 Đề thi học sinh giỏi môn toán khối 10 khu vực DUYÊN HẢI VÀ ĐỒNG BẰNG BẮC BỘ n...

Đã gửi bởi Gachdptrai12 on 24-04-2016 - 21:30 trong Thi HSG cấp Tỉnh, Thành phố. Olympic 30-4. Đề thi và kiểm tra đội tuyển các cấp.

Mình có cách "sơ cấp" hơn :)

Đặt $x=\frac{1}{a};y=\frac{1}{b};z=\frac{1}{c}$ thì từ giả thiết ta được $xy+yz+xz=1$.

Yêu cầu bài toán viết lại thành:

$3+\sum \frac{x^2}{y}\geq (x+y+z)^2+\sqrt{3}$

Mặt khác: $\sum \frac{x^2}{y}=(\sum xy) (\sum \frac{x^2}{y})\geq (x+y+z)(x^2+y^2+z^2)$ nên ta chỉ cần chứng minh:

$p^3-p^2-2p+3-\sqrt{3}\geq 0$ với $p=x+y+z\geq \sqrt{3}$.

Chứng minh được BĐT này đúng :)

ko biết giải bằng cách ni có đúng ko :)

bđt <=> f(r)=3+$\frac{x^{3}y+y^{3}z+z^{3}x}{r}$-p2-$\sqrt{3}$ ta có dễ thấy f(r) là hàm đơn điệu theo r nên nó đạt Min khi có 2 biến bằng nhau là đủ như vậy ta thấy giả sử x=y thì điều kiên được viết lại x2+zx=1 và bđt trở thành $x+\frac{2x^{2}}{z}+3-(2x+z)^{2}-\sqrt{3}\geq 0$

rút thế tách về biểu thức 1 ẩn z thì ta sẽ chứng minh bđt 1 biến theo f(z) 
P/s ai có thể dùng AM-GM 1 cách thuần túy cho bài ni luôn ko :v :v




#629279 3+\frac{a^{2}}{b}+\frac{b^{...

Đã gửi bởi Gachdptrai12 on 24-04-2016 - 10:16 trong Bất đẳng thức - Cực trị

Đề Duyên Hải Bắc Bộ 

cho a,b,c dương thỏa a+b+c=abc chứng minh 

$3+$\frac{a}{b^{2}}+\frac{b}{c^{2}}+\frac{c}{a^{2}}$ \geq (\sum \frac{1}{a})^{2}+\sqrt{3}$




#629116 $\frac{1}{2a^{2}+bc}+\frac{...

Đã gửi bởi Gachdptrai12 on 23-04-2016 - 17:47 trong Bất đẳng thức - Cực trị

sai la

 

sao lại đúng được nếu a+b-2c$\leq$0 thì sao

bạn ơi bài này có tính hoán vị ta có thể giả sử c= min(a,b,c)  :D  :D  :icon10:  :icon10:  vì thế mà a+b>=2c mà thôi :v 




#629074 $\frac{1}{2a^{2}+bc}+\frac{...

Đã gửi bởi Gachdptrai12 on 23-04-2016 - 11:41 trong Bất đẳng thức - Cực trị

Giả sử $a \geqslant b \geqslant c.$ Áp dụng bất đẳng thức Cauchy Schwarz, ta có \[\sum  \frac{1}{2a^{2}+bc} = \frac{1}{{2{a^2} + bc}} + \frac{1}{{2{b^2} + ca}} + \frac{1}{{4{c^2} + 2ab}} + \frac{1}{{4{c^2} + 2ab}} \ge \frac{{16}}{{2(a+b)^2 + 8{c^2} + c(a + b)}}.\] Như vậy ta chỉ cần chứng minh \[\frac{{16}}{{2(a+b)^2 + 8{c^2} + c(a + b)}} \ge \frac{8}{{{{(a + b + c)}^2}}},\] tương đương với \[2{(a + b + c)^2} \ge 2(a+b)^2 + 8{c^2} + c(a + b),\] hoặc \[3c(a + b - 2c) \ge 0.\] Hiển nhiên đúng.

a Huyện có thể chứng minh cái bđt ni là bài bđt được chứng minh 

$(a+b+c)^{4}\geq 2(\sum (b+c)^{2}(2a^{2}+bc))$




#629035 $\frac{1}{2a^{2}+bc}+\frac{...

Đã gửi bởi Gachdptrai12 on 22-04-2016 - 22:33 trong Bất đẳng thức - Cực trị

cho a,b,c là các số thực ko âm thỏa ab+bc+ca >0 chứng minh 

$\frac{1}{2a^{2}+bc}+\frac{1}{2b^{2}+ca}+\frac{1}{2c^{2}+ab}\geq \frac{8}{(a+b+c)^{2}}$




#628863 $EG$ vuông góc $GF$

Đã gửi bởi Gachdptrai12 on 21-04-2016 - 23:13 trong Hình học

Cho đoạn thẳng $BC. E$ là trung điểm $BC,D$ là trung điểm $EC$. Trên trung trực đoạn $CE$ lấy $F$ bất kỳ. $A$ thuộc tia đối tia $FC$ sao cho $FC=3FA. (AEC)$ giao $AB$ lần hai tại $G$. Chứng minh $EG$ vuông góc $GF$.




#628541 $\sqrt{\frac{a}{b+c}}+\sqrt...

Đã gửi bởi Gachdptrai12 on 20-04-2016 - 19:22 trong Bất đẳng thức - Cực trị

 Câu 1 sử dụng đánh giá $\sqrt{\dfrac{a}{b+c}}\geq \sqrt{\dfrac{a^2}{ab+bc+ca}}$ xong áp dụng AM-GM là ra

 Câu 2 thì sai đề rồi  :luoi:

đã sửa 




#628387 $\sqrt{\frac{a}{b+c}}+\sqrt...

Đã gửi bởi Gachdptrai12 on 19-04-2016 - 23:29 trong Bất đẳng thức - Cực trị

1)cho a,b,c là các số thực ko âm chứng minh 

$\sqrt{\frac{a}{b+c}}+\sqrt{\frac{b}{c+a}}+\sqrt{\frac{c}{a+b}}+\frac{9\sqrt{ab+bc+ca}}{a+b+c}\geq 6$(bài ni dấu bằng khá trí )

2)cho a,b,c dương chứng minh

(a+b)2(b+c)2(c+a)2(ab+bc+ca)2$\geq$ 8a2b2c2(a2+b2+c2)2




#627869 $\frac{ab}{3a^{2}+b^{2}}+...

Đã gửi bởi Gachdptrai12 on 17-04-2016 - 22:25 trong Bất đẳng thức - Cực trị

Chứng minh gì vậy bạn?

đã sửa 




#627720 $\frac{ab}{3a^{2}+b^{2}}+...

Đã gửi bởi Gachdptrai12 on 17-04-2016 - 13:59 trong Bất đẳng thức - Cực trị

cho a,b,c dương chứng minh
$\frac{ab}{3a^{2}+b^{2}}+\frac{bc}{3b^{2}+c^{2}}+\frac{ca}{3c^{2}+a^{2}}$$\leq \frac{3}{4}$



#627210 Topic tập hợp các BĐT đánh giá từng biến

Đã gửi bởi Gachdptrai12 on 15-04-2016 - 09:26 trong Bất đẳng thức - Cực trị

 

Đề bài 6

 

Cho x, y và z là ba số thực dương, thoả mãn:

 

$x^{2}+y^{2}+z^{2}=2\left ( xy+yz+zx \right ).$
 
Chứng minh rằng:
 
$\frac{x}{y}+\frac{y}{z}+\frac{z}{x}\geq 5.$

 

bài ni có 1 ý tưởng đó là dùng bổ đề hoán vị nhưng thấy rắc rối quá và cái bổ đề cũng khó chứng minh nữa :3 @@ ta cũng có thể tổng quát lên thành đề VMEO tháng 11 THPT :)))

p/s ai có thể dùng phân tích p,q,r giải dc ko vại 

một bài toán thú vị khác

 

Bài 7:  Cho n số thực$ x_1,x_2,...,x_n thoả mãn x_1+x_2+...+x_n=n$. Đặt :

 
$x_1^2+x_2^2+...+x_n^2=n+n(n-1)t^2$
 
với $t\geq 0$. Khi đó ta có :
 
$1-(n-1)t\leq x_i\leq 1+(n-1)t,\;\forall i=\overline{1,n}$



#626046 Inequalities From 2016 Mathematical Olympiads

Đã gửi bởi Gachdptrai12 on 09-04-2016 - 08:54 trong Bất đẳng thức - Cực trị

Dòng màu đỏ cần có đk $a,b,c >0$ để nó có nghĩa.

Ta có $a^3 +b^3+c^3 -3abc =(a+b+c)(\frac{(a-b)^2+(b-c)^2+(c-a)^2}{2} \geq 0 \Leftrightarrow a+b+c \geq 0$
Mà $\sum \sqrt[6]{x^2 +\frac{1}{y^2}} > 0$ nên $\sum \sqrt{x^2+\frac{1}{y^2}} \geq 3\sqrt[6]{(x^2+\frac{1}{y^2})(y^2+\frac{1}{z^2})(z^2+\frac{1}{x^2})}$.
Áp dụng Holder: $(x^2+\frac{1}{y^2})(y^2+\frac{1}{z^2})(z^2+\frac{1}{x^2}) \geq (\sqrt[3]{(xyz)^2} +\sqrt[3]{\frac{1}{(xyz)^2}})^3$.
$\Leftrightarrow 3\sqrt[6]{(x^2+\frac{1}{y^2})(y^2+\frac{1}{z^2})(z^2+\frac{1}{x^2})} \geq 3\sqrt{\sqrt[3]{(xyz)^2} +\sqrt[3]{\frac{1}{(xyz)^2}}}$
Cần chứng minh $3\sqrt{\sqrt[3]{(xyz)^2} +\sqrt[3]{\frac{1}{(xyz)^2}}} \geq 3\sqrt{2}$
hay $\sqrt[3]{(xyz)^2} +\sqrt[3]{\frac{1}{(xyz)^2}} \geq 2$ (điều này đúng theo AM - GM)
Dấu $"="$ xảy ra khi $x=y=z=1$ hoặc $x=y=z=-1$

dòng 1 bđt sai nếu a,b chạy tơí âm vô cùng còn c=0